Practice Questions Flashcards
What percentage of the full Social Security benefit will a worker receive if he or she retires 9 months prior to attaining the individual's full Social Security retirement age? (A) 50 percent of total benefit (B) 80 percent of total benefit (C) 95 percent of total benefit (D) 100 percent of total benefit
The answer is (C). The benefit is reduced by 5/9 of one percent for each month prior to the full Social Security retirement age (5/9 of 1% x 9 = 5%). (A), (B), and (D) do not use the correct formula. (Chapter 19)
An employee exercises nonqualified stock options with an option price of $5 a share and a market price of $10 a share. How much ordinary income does the individual have for each share at the time of exercise? (A) $ 0 (B) $ 5 (C) $ 10 (D) $ 15
The answer is (B). (A), (C), and (D) are incorrect. With a nonqualified stock option, the participant receives ordinary income in the amount of the difference between the market value at the time of exercise and the option price ($10 - $5 = $5). (Chapter 16)
Which of the following statements concerning the SIMPLE is correct?
(A) To sponsor a SIMPLE, an employer must have 25 or fewer employees.
(B) An employer can sponsor both a SIMPLE and a SEP.
(C) An employer cannot place any restrictions on participant withdrawals.
(D) The deferral limit for a SIMPLE is the same as for a 401(k) plan.
The answer is (C). (A) is incorrect because the maximum number of employees an employer can have and still sponsor a SIMPLE is 100, not 25. (B) is incorrect because a SIMPLE sponsor cannot sponsor any other tax-advantaged retirement plan, including qualified plans, SEPs, and 403(b) plans. (D) is incorrect because the maximum employee salary deferral in a SIMPLE is lower than in a 401(k) plan. (Chapter 6)
Which of the following statements concerning contributions to a SIMPLE is correct?
(A) The employer can elect to provide a 50-cent match for each dollar that the employee elects to defer, as long as the employer matches a salary deferral of up to 6 percent of compensation.
(B) Employee salary deferrals are subject to a nondiscrimination test.
(C) A contribution for all eligible employees in the amount of 2 percent of compensation satisfies the employer-contribution requirement.
(D) The employer has the option to skip making contributions for a specific year.
The answer is (C). (A) is incorrect because the employer-matching contribution to a SIMPLE must be a dollar-for-dollar match up to 3 percent that employees elect to defer. (B) is incorrect because employees can make pretax salary deferrals of up to the maximum deferral limit without regard to how much the other employees contribute. (D) is incorrect because the employer must always contribute either a matching contribution or a nonelective contribution. (Chapter 6)
Which of the following statements concerning employee contributions to a qualified plan (other than to a 401(k) plan) is correct?
(A) Employee contributions to a plan must comply with the top-heavy vesting schedule.
(B) Voluntary employee contributions must satisfy a nondiscrimination test.
(C) Contributions of up to 50 percent of pay can be made if a plan allows voluntary employee contributions.
(D) Voluntary contributions are not counted as annual additions under the limit of Code Sec. 415c. (C).
The answer is (B). Voluntary contributions must satisfy the ACP test. Essentially, this means that highly compensated employees cannot make such contributions unless nonhighly compensated employees contribute to the plan. (A) is incorrect because voluntary contributions must be fully vested at all times. (C) is incorrect because it is a nonexistent requirement. (D) is incorrect because voluntary contributions are treated as annual additions. (Chapter 8)
The systematic withdrawal strategy to retirement income
(A) relies heavily on annuities, bonds, and guarantee income streams to create a systematic income stream to meet basic expenses every year.
(B) often relies on market investments like stocks in order to generate investment growth to help meet retirement income needs.
(C) does not allow for a retiree to be invested in bonds, CDs, or annuities due to their low yields.
(D) cannot provide for flexible spending behaviors of the client nor can it adjust the income to deal with inflation.
(B) is correct
(A) is incorrect because this is describing the flooring approach not the systematic withdrawal approach. (C) is incorrect because the systematic withdrawal approach takes an agnostic view of products, and will utilize all of these investment options if it can help improve total returns for the portfolio through diversification and improved safety of investments. (D) is incorrect because systematic withdrawal strategies can be built to increase income over time to handle inflation or variable spending needs.
If a defined-benefit plan covered by the PBGC has sufficient assets to pay the present value of accrued benefits, the plan qualifies for a (A) standard termination (B) distress termination (C) abandoned plan termination (D) simple termination
standard termination
Which of the following statements concerning a stock option program is correct?
(A) The employer does not ever get a deduction with a nonqualified stock option program.
(B) A nonqualified stock option program must be available for all employees.
(C) With an incentive stock option (ISO) program, the participants must meet certain holding period requirements to take full advantage of the special tax rules.
(D) There are never any income tax consequences at the time participants sell stock within a nonqualifed stock option program.
The answer is (C). (A) is incorrect because, at the time of exercise, the employer gets a deduction in the amount that the participant has in ordinary income. (B) is incorrect because, like other programs of executive compensation, the program can be only for a small group of executives. (D) is incorrect because, at the time of the sale, the participant has long-term capital gain if the sale price exceeds the market value at the time of exercise. (Chapter 16)
Which of the following statements concerning defined-contribution plans with contribution formulas that are integrated with Social Security is correct?
(A) Only money-purchase pension plans can have a formula that integrates with Social Security.
(B) A plan with a formula that integrates with Social Security must be tested every year to ensure the plan does not violate the 401a. (A)(4) requirements.
(C) The maximum integration level is covered compensation.
(D) If the integration level is the taxable wage base and employees receive a contribution of 3 percent of total compensation, an additional 3 percent can be contributed for compensation in excess of the taxable wage base.
The answer is (D). (A) is incorrect because all types of defined-contribution plans can be integrated with Social Security. (B) is incorrect because properly integrated formulas satisfy a design safe harbor. This means that nondiscrimination tests do not have to be performed each year. (C) is incorrect because the maximum integration level in a defined-contribution plan is the current year’s taxable wage base. Covered compensation is typically the integration level in a defined-benefit plan. (Chapter 8)
Which of the following statements concerning the Code Sec. 401a. (A)(4) nondiscrimination rules that apply to qualified plans is correct?
(A) The regulations require that all plans perform a mathematical test each year.
(B) The regulations confirm that a plan satisfies the rules if highly compensated employees receive contributions that are no more than two times the rate of contribution for nonhighly compensated employees.
(C) There are only subjective “fact and circumstances” tests for determining whether a plan’s benefit or contribution formula satisfies the nondiscrimination rules.
(D) A plan can show that it satisfies the nondiscrimination rules by demonstrating that either benefits or contributions are nondiscriminatory.
The answer is (D). This is a correct statement of the rules. (A) is incorrect because a plan that satisfies one of the design safe-harbors does not have to do any other testing. (B) is incorrect because there is no such rule. The rules do say that if contributions constitute a level percentage of compensation for all employees, then the plan will be in compliance. (C) is incorrect because there are clear objective tests under the regulations for determining whether a plan satisfies the rules. (Chapter 8)
For taxpayers under age 70½ who are active participants in an employer-maintained qualified plan, which of the following statements concerning annual deductible contributions to an individual retirement account (IRA) in 2017 is correct?
(A) For unmarried taxpayers, a full IRA deduction is allowed for adjusted gross income of up to $62,000.
(B) For married taxpayers filing jointly, a full IRA deduction is allowed for adjusted gross income of up to $119,000.
(C) For unmarried taxpayers, a reduced deduction is allowed for adjusted gross income of up to $150,000.
(D) These taxpayers are ineligible to make any tax-deductible contributions to an IRA.
The answer is (A). Table 17-1 in the textbook summarize all of the applicable rules. (B), (C), and (D) are incorrect because they do not correctly state the IRA phaseout limits. (Chapter 17)
Esther is a participant in her employer’s profit-sharing plan. Esther attained age 70 on October 1, 2014. She is not a 5 percent owner and she retires on January 15, 2017. Her required beginning date is
(A) April 1, 2015
(B) April 1, 2016
(C) April 1, 2017
(D) April 1, 2018
The answer is (D). Because this is a qualified plan, Esther is not a 5 percent owner, and because she continues working beyond age 70½, the required beginning date is the April 1 following the calendar year in which she retires. (Chapter 24)
Which of the following statements concerning participant loans from qualified retirement plans is correct?
(A) A qualified plan must include a participant loan provision in one form or another.
(B) Nonhighly compensated participants typically are charged below-market interest rates.
(C) Loans frequently are secured by using a participant’s account balance as security.
(D) Loans can be limited only to the owners of the company.
The answer is (C). Securing the loan with the account balance is a good idea because it both satisfies the DOL’s regulations and simplifies the administration of the loan. (A) is incorrect because a plan need not include a loan provision. (B) is incorrect because the plan is required to charge a market rate of interest. (D) is incorrect because a loan program must be available to all participants. (Chapter 9)
Which of the following statements concerning Code Sec. 409A that applies to nonqualified deferred-compensation plans is correct?
(A) Deferral amounts not subject to a substantial risk of forfeiture that do not comply with the distribution provisions are subject to a 100 percent penalty tax.
(B) Payments to meet the requirements of a domestic relations order do not violate the acceleration of benefit provisions.
(C) A distribution to pay for a child’s college education is likely to be considered an unforeseeable emergency.
(D) A participant will generally be able to make a benefit election at the time distributions are to be made from the plan.
The answer is (B). This is an exception to the general rule that benefits cannot be accelerated. (A) is incorrect because the penalty tax is 20 percent. (C) is incorrect because regulations specifically identify this as a situation that is not an unforeseeable emergency. (D) is incorrect because elections have to be made at the time of the deferral election, not the time of distribution. (Chapter 15)
Which of the following statements concerning the reversion of assets in a qualified plan is correct?
(A) There is a 50 percent excise tax if the entire amount of the excess assets in a defined-benefit plan is reverted to the employer.
(B) A reversion can occur in a defined-contribution plan if assets exceed promised assets.
(C) A defined-benefit plan can be amended at the time of termination to allow for a reversion of plan assets.
(D) There is no excise tax if 20 percent of the excess assets is shared with participants in a defined-benefit plan.
The answer is (A). The excise tax is a prohibitive 50 percent unless a portion of the excess is shared with participants. (B) is incorrect because a defined-contribution plan does not have excess assets. All assets are allocated among the participants. (C) is incorrect because a defined-benefit plan must provide for a reversion at the time the plan is drafted. It cannot be amended later to allow a reversion. (D) is incorrect because the tax does not go away if a portion of the excess is shared with participants. The tax is reduced to 20 percent. (Chapter 14)
As far as profit-sharing plans are concerned, the IRS will recognize an involuntary termination under which of the following circumstances?
(A) The minimum-funding standards have not been satisfied.
(B) The plan contributions are not substantial and recurring.
(C) The long-run liability of the company to the PBGC is expected to increase unreasonably.
(D) A substantial owner has terminated with a plan distribution of over $10,000 and the plan is left with unfunded vested liabilities.
The answer is (B). In the case of profit-sharing-type plans, failure to make substantial and recurring contributions results in an involuntary plan termination as of the time contributions cease. Participants become fully vested in their benefits as of the date of the involuntary plan termination. (A) and (C) are incorrect because profit-sharing plans are not subject to actuarial funding. (D) is incorrect because other restrictions apply. (Chapter 14)
Jim Rock, aged 65, is retired and annually receives a fully taxable $29,000 pension income and $9,000 in Social Security benefits. Jim has no other sources of retirement income and files jointly with his wife. How much of Jim's Social Security benefits will be subject to federal income taxation given his provisional income? (A) $0 (B) $ 750 (C) $ 1,500 (D) $ 9,000
The answer is (B). Because Jim Rock’s adjusted gross income ($29,000) plus one-half of his Social Security income ($9,000/2 = $4,500) exceeds $32,000 (but does not exceed $44,000), as a married taxpayer, he will have to include $750, or one-half of the excess over the $32,000 threshold, as taxable income ($33,500 - $32,000 = 1,500/2 = $750). (A), (C), and (D) do not use the correct formula. (Chapter 19)
Which of the following statements concerning a Sec. 423 employee stock purchase plan is correct?
(A) The effective discount can be substantially higher than 15 percent off the stock’s price.
(B) The plan can be limited to a few highly compensated executives.
(C) Participants pay taxes at the time the stock is purchased.
(D) Like nonqualified stock options, the entire value of the discount is taxed as ordinary income.
The answer is (A). If the price of the stock rises over the purchase period and the price is 15 percent off the market price at the beginning of the period, the discount can vastly exceed 15 percent of the market price of the stock at the time of the purchase. (B) is incorrect because these plans must cover most full-time employees. (C) is incorrect because taxes are not paid until the stock is sold. (D) is incorrect because only a portion of the value is taxed as ordinary income. (Chapter 16)
Which of the following are benefits typically covered under Part A of Medicare?
hospital nursing services
Which of the following statements concerning the “retirement ladder” model of retirement savings is correct?
(A) Personal savings has replaced Social Security benefits in the retirement ladder.
(B) Home equity is a possible source of funding for retirement that was not included on the old three-legged stool.
(C) Inheritances are a key retirement source because about 80 percent of baby boomers will be receiving significant inheritances.
(D) Wages are not included as a source of retirement funding because an individual cannot be retired and still work.
The answer is (B). Home equity was not recognized in the “three-legged stool” model of sources of retirement income, but it has been added to the expanded list included in the retirement ladder. (A) is incorrect because the retirement ladder still considers Social Security, personal savings, and employer pensions as key sources of retirement funding. (C) is incorrect because only about 15 percent of baby boomers expect significant inheritances. (D) is incorrect because many individuals who consider themselves retired will choose to do some form of work. (Chapter 20)
Retirement Ladder
Financial Independence in Retirement! Other forms of support Inheritances Retiree health, long-term care, and other insurance solutions Personal savings Informed planning Fiscal welfare/social assistance Part-time wages Company-sponsored retirement plans Social Security Home equity life insurance Rental income
Which of the following documents must be provided automatically to all participants in a qualified plan?
summary plan description
The annual report is filed with the government and is not furnished to participants, although they may request to see it. Participants have a right to see and copy the plan document and trust agreement. However, it need not be provided automatically to them.
Which of the following statements concerning the fee disclosure rules that apply to service providers is correct?
(A) The rules apply to SEPs and SIMPLEs.
(B) The rules only require that service providers report direct compensation.
(C) If the rules are not followed the service providers are charged a $1,000 fine.
(D) The rules apply to those providing brokerage services to a 401(k) plan with participant-directed accounts.
The answer is (D). (A) is incorrect because the fee disclosure rules do not apply to SEPs and SIMPLEs. (B) is incorrect because covered service providers must disclose both direct and indirect compensation. (C) is incorrect because if the rules are not followed the fiduciary has engaged in a prohibited transaction and the service provider can be subject to a penalty tax, which is 15 percent of the amount involved. (Chapter 12)
Which of the following statements concerning the minimum funding requirements is correct?
(A) Plans that are “at risk” are given additional time to fund the plan.
(B) Fully insured (Code Sec. 412(e)) plans are subject to additional minimum funding requirements.
(C) The funding target normal cost is the cost for the current year’s benefit accruals.
(D) The rules allow a plan sponsor to choose from a wide range of actuarial cost methods.
The answer is (C). (A) is incorrect because at-risk plans are subject to more accelerated funding. (B) is incorrect because fully insured plans are exempt from the minimum funding requirements (D) is incorrect because the rules today require all sponsors to use the same method for determining costs; in the past employer’s had more choice. (Chapter 11)